USAMO Problem 2

What is the smallest integer n n , greater than one, for which the root-mean-square of the first n n positive integers is an integer?

N o t e . \mathbf{Note.} The root-mean-square of n n numbers a 1 , a 2 , , a n a_1, a_2, \cdots, a_n is defined to be [ a 1 2 + a 2 2 + + a n 2 n ] 1 / 2 \left[\frac{a_1^2 + a_2^2 + \cdots + a_n^2}n\right]^{1/2}

This problem is part of this set .


The answer is 337.

This section requires Javascript.
You are seeing this because something didn't load right. We suggest you, (a) try refreshing the page, (b) enabling javascript if it is disabled on your browser and, finally, (c) loading the non-javascript version of this page . We're sorry about the hassle.

1 solution

The sum of the first n n consecutive squares is known as the n n 'th pyramid number. The formula for such a number is P n = n ( n + 1 ) ( 2 n + 1 ) 6 P_n = \frac{n(n+1)(2n+1)}{6}

So the problem we are asked to solve corresponds to solving the equation P n n = y \sqrt{\frac{P_n}{n}} = y where y y is some positive integer.

Notice that we can rewrite the equation above as P n n = y 1 6 ( n + 1 ) ( 2 n + 1 ) = y 2 2 n 2 + 3 n + 1 = 6 y 2 2 ( n + 3 4 ) 2 1 8 = 6 y 2 ( 4 n + 3 ) 2 48 y 2 = 1 \begin{aligned} \sqrt{\frac{P_n}{n}} & = y \Rightarrow \\ \frac{1}{6}(n+1)(2n+1) & = y^2 \Rightarrow \\ 2n^2+3n+1 & = 6y^2 \Rightarrow \\ 2\left(n+\frac{3}{4}\right)^2 - \frac{1}{8} & = 6y^2 \Rightarrow \\ (4n+3)^2 - 48y^2 & = 1 \end{aligned} If we let x = 4 n + 3 x = 4n + 3 then we are looking for solutions to the Pell equation for which x 3 ( m o d 4 ) x \equiv 3 \pmod{4} . For a moment, leave aside the extra restriction on x x .

It is easy to see that ( x , y ) = ( 7 , 1 ) (x, y) = (7, 1) is a solution to x 2 48 y 2 = 1 x^2 - 48y^2 = 1 . Observe that if ( x 0 , y 0 ) (x_0, y_0) is a solution then computing ( x 0 + 48 y 0 ) k = x k + 48 y k (x_0 + \sqrt{48} y_0 )^k = x_k + \sqrt{48} y_k yields another solution ( x k , y k ) ( x_k, y_k ) for any k k .

A few hand calculations generate the following solutions ( 7 , 1 ) , ( 97 , 14 ) , ( 1351 , 195 ) , ( 18817 , 2716 ) , (7, 1), (97, 14), (1351, 195), (18817, 2716), \ldots The least ''non-trivial'' x x -value that is congruent to 3 modulo 4 is x = 1351 x = 1351 . Thus, n = 1351 3 4 = 337 n = \frac{1351 - 3}{4} = \boxed{337}

Oh damn got till that Pell's equation(not by this method though) but just forgot that formula right now(ah darn me) for its solution so had to use programming. Thanks for the Pell's equations

Kartik Sharma - 6 years, 2 months ago

Log in to reply

What's the proof? Had to do something with generating functions?

Kartik Sharma - 6 years, 2 months ago

Log in to reply

Without going into details, say we have a solution ( x 0 , y 0 ) (x_0, y_0) , in our case the minimal solution, then let us rewrite the following x k 2 48 y k 2 = ( x 0 2 48 y 0 2 ) k = 1 ( x k + 48 y k ) ( x k 48 y k ) = ( x 0 + 48 y 0 ) k ( x 0 48 y 0 ) k \begin{aligned} x_k^2 - 48 y_k^2 & = (x_0^2 - 48 y_0^2)^k = 1 \Rightarrow \\ (x_k + \sqrt{48} y_k)(x_k - \sqrt{48} y_k) & = (x_0 + \sqrt{48} y_0)^k (x_0 - \sqrt{48} y_0)^k \end{aligned} So x k + 48 y k = ( x 0 + 48 y 0 ) k x k 48 y k = ( x 0 4 48 y 0 ) k x k + 48 y k + ( x k 48 y k ) = ( x 0 + 4 3 y 0 ) k + ( x 0 4 3 y 0 ) k x k = ( x 0 + 4 3 y 0 ) k + ( x 0 4 3 y 0 ) k 2 \begin{aligned} x_k + \sqrt{48} y_k & = (x_0 + \sqrt{48} y_0)^k \\ x_k - \sqrt{48} y_k & = (x_0 - 4\sqrt{48} y_0)^k \Rightarrow \\ x_k + \sqrt{48} y_k + (x_k - \sqrt{48} y_k) & = (x_0 + 4\sqrt{3} y_0)^k + (x_0 - 4\sqrt{3} y_0)^k \\ \Rightarrow x_k & = \frac{(x_0 + 4\sqrt{3} y_0)^k + (x_0 - 4\sqrt{3} y_0)^k}{2} \end{aligned} We can do similarly for y k y_k .

The formula above is not the most convenient way of generating solutions by hand. I have revised my solution a bit such that it applies a slightly easier technique to generate solutions.

Martin Sergio H. Faester - 6 years, 1 month ago

Nice! Took me a while to notice the Pell equation.

Jake Lai - 6 years, 2 months ago

0 pending reports

×

Problem Loading...

Note Loading...

Set Loading...